Find the angle of depression
from point A to point C.
?
66°
5 mi
B
Angle of depression = [ ? ]

Find The Angle Of Depressionfrom Point A To Point C.?665 MiBAngle Of Depression = [ ? ]

Answers

Answer 1

Given:

In triangle ABC, [tex]m\angle A=66^\circ, AB=5\text{ mi}[/tex].

To find:

The angle of depression from point A to point C.

Solution:

According to angle sum property, the sum of all interior angles of a triangle is 180 degrees.

In triangle ABC,

[tex]m\angle A+m\angle B+m\angle C=180^\circ[/tex]

[tex]66^\circ+90^\circ+m\angle C=180^\circ[/tex]

[tex]156^\circ+m\angle C=180^\circ[/tex]

[tex]m\angle C=180^\circ-156^\circ[/tex]

[tex]m\angle C=24^\circ[/tex]

We know that if a transversal line intersect the two parallel lines, then alternate interior angles are equal. So, the angle of depression from point A to point C is equal to the measure of angle C in triangle ABC.

Therefore, the angle of depression is 24 degrees.


Related Questions

el que me responda rápido le doy coronita cuanto es 99+88+77+66+55+44+33+22+11+11+22+33+44+55+66+77+88+99

Answers

Answer:

990

Step-by-step explanation:

99+88+77+66+55+44+33+22+11+11+22+33+44+55+66+77+88+99=990

Answer:

Answer:

990

Step-by-step explanation:

99+88+77+66+55+44+33+22+11+11+22+33+44+55+66+77+88+99=990

Step-by-step explanation:

A fuel pump at a gasoline station doesn't always dispense the exact amount displayed on the meter. When the
meter reads 1.000 L, the amount of fuel a certain pump dispenses is normally distributed with a mean of 1 L
and standard deviation of 0.05 L. Let X = the amount dispensed in a random trial when the meter reads
1.000 L
Find P(X < 1).

Answers

Answer:

P(X < 1) = 0.5

Step-by-step explanation:

Normal Probability Distribution:

Problems of normal distributions can be solved using the z-score formula.

In a set with mean [tex]\mu[/tex] and standard deviation [tex]\sigma[/tex], the z-score of a measure X is given by:

[tex]Z = \frac{X - \mu}{\sigma}[/tex]

The Z-score measures how many standard deviations the measure is from the mean. After finding the Z-score, we look at the z-score table and find the p-value associated with this z-score. This p-value is the probability that the value of the measure is smaller than X, that is, the percentile of X. Subtracting 1 by the p-value, we get the probability that the value of the measure is greater than X.

Mean of 1 L and standard deviation of 0.05 L.

This means that [tex]\mu = 1, \sigma = 0.05[/tex]

Find P(X < 1).

This is the p-value of Z when X = 1. So

[tex]Z = \frac{X - \mu}{\sigma}[/tex]

[tex]Z = \frac{1 - 1}{0.05}[/tex]

[tex]Z = 0[/tex]

[tex]Z = 0[/tex] has a p-value of 0.5. Thus

P(X < 1) = 0.5

Let P(x, y) denote the point where the terminal side of an angle θ meets the unit circle. If P is in Quadrant II and x = − 5⁄8 , evaluate the six trigonometric functions of θ.

Answers

The six trigonometric function of [tex]\theta[/tex] are [tex]\sin \theta \approx 0.781[/tex], [tex]\cos \theta = - \frac{5}{8}[/tex], [tex]\tan \theta \approx -1.250[/tex], [tex]\cot \theta \approx -0.800[/tex], [tex]\sec \theta = - \frac{8}{5}[/tex], [tex]\csc \theta \approx 1.280[/tex], respectively.

In this question, we assume that x-component of the terminal point is part of a unit circle. Then, we can find the value of y by means of the Pythagorean theorem:

[tex]y = \sqrt{1-x^{2}}[/tex] (1)

If we know that [tex]x = -\frac{5}{8}[/tex] and P is in the second quadrant, then the value of y is:

[tex]y = + \sqrt{1-\left(-\frac{5}{8} \right)^{2}}[/tex]

[tex]y \approx 0.781[/tex]

By trigonometry, we remember the following definitions for the six basic trigonometric functions:

[tex]\sin \theta = \frac{y}{1}[/tex] (1)

[tex]\cos \theta = \frac{x}{1}[/tex] (2)

[tex]\tan \theta = \frac{y}{x}[/tex] (3)

[tex]\cot \theta = \frac{1}{\tan\theta}[/tex] (4)

[tex]\sec \theta = \frac{1}{\cos \theta }[/tex] (5)

[tex]\csc \theta = \frac{1}{\sin \theta}[/tex] (6)

If we know that [tex]x = -\frac{5}{8}[/tex] and [tex]y \approx 0.781[/tex], then the six basic trigonometric functions are, respectively:

[tex]\sin \theta \approx 0.781[/tex], [tex]\cos \theta = - \frac{5}{8}[/tex], [tex]\tan \theta \approx -1.250[/tex], [tex]\cot \theta \approx -0.800[/tex], [tex]\sec \theta = - \frac{8}{5}[/tex], [tex]\csc \theta \approx 1.280[/tex]

The six trigonometric function of [tex]\theta[/tex] are [tex]\sin \theta \approx 0.781[/tex], [tex]\cos \theta = - \frac{5}{8}[/tex], [tex]\tan \theta \approx -1.250[/tex], [tex]\cot \theta \approx -0.800[/tex], [tex]\sec \theta = - \frac{8}{5}[/tex], [tex]\csc \theta \approx 1.280[/tex], respectively.

We kindly invite you to check this question related to trigonometric functions: https://brainly.com/question/6904750

TERCER TRIMESTRE SEGUNDO GRADO
DEL 7 DE JUNIO AL 18 DE JUNIO
4- Calcular la media, la mediana y la moda de la siguiente serie de numeros: 5, 3, 6, 5, 4, 5, 2, 8, 6,5,4,8,3,4,5,4,8,2
5.4.

Answers

Answer:

MEDIA: 4.8

MEDIANA: 5

MODA: 5

espero que te ayude :)

NEED HELP DUE IN 2 MINUTESS!!!!

Answers

Answer:

17.83 in

Step-by-step explanation:

2 x 3.14

= 6.28

112 ÷ 6.28= 17.8343949

Whole number is 17.83 in

The correct answer would be 17.83in

Huilan is 11 years older than Thomas. The sum of their ages is 67 . What is Thomas's age?

Answers

Answer:

Thomas is 28.

Step-by-step explanation:

Let Huilan’s age be “x + 11”

Let Thomas’s age be “x”

x + x + 11 = 67

2x + 11 = 67

2x = 56

x = 28 years

x + 11 = 39 years

Thomas is 28.

5th grade math. correct answer will be marked brainliest

Answers

Answer:

I put 6/9 even though i know its wrong

Step-by-step explanation:

Can someone please explain to me a easier way of understanding this please

Answers

Step-by-step explanation:

u have solve it by finding slope(m)

and substitute is general eq of straight line

y = mx + c

Answer:

y = 2x - 6

Step-by-step explanation:

These are actually pretty fun to do once you get the hang of it! Here are the steps!

The formula for slope intercept form is y=mx + b.

m is the slope and b is the y-intercept. Let's start with finding the y-intercept (also known as b).

To find the y-intercept, look at where the line crosses the y-axis. In your equation, the line crosses the y-axis at -6. Plug -6 in for b in the equation.

y = mx + (-6)

Now were going to find the slope. To find the slope, you need to find the rise over run. To start, first find a point on the graph where the line goes through an integer (just pick one of the blue dots). Then, count how many squares you go up or down, and how many you go to the left or right before you hit the next blue dot. Divide the rise over the run, and you have your slope! Here are the steps for the problem:

To get from one dot to another, you rise up 2 units, and you run to the right 1 unit. Plugging those integers into rise/run, you get 2/1 which simplifies to 2. That's your slope.

Now all we need to do is plug the slope into the formula, and you have your equation!

Answer: y = 2x - 6

Find the 7th term of the geometric sequence
7, 24.5, 85.75, ...

Answers

Answer:

12,867.859375

Step-by-step explanation:

24.5/7 = 3.5

Geometric sequence

A_n = 7 * (3.5)^(n - 1)

A_7 = 7 * (3.5)^(7 - 1)

A_7 = 12,867.859375

Which of the following will most likely produce sample proportions that are
normally distributed?
A. Many samples of 34 coin flips
B. Many samples of 14 coin flips
C. Many samples of 19 coin flips
D. Many samples of 9 coin flips

Answers

The answer you are looking for is 14.

Help!!!! 20 points!!!!!!’

Answers

Answer:

option c is the correct answer

first take y raised to the power 4 common then cancel its power by y raised to the power 3

you get your answer

The graph of y=h(x) is a line segment joining the points (1, -5) and (9,1).
Drag the endpoints of the segment below to graph y = h-'(x).

Answers

Answer:

[tex]h^{-1}(x) = \frac{4}{3}x + \frac{23}{3}[/tex]

Step-by-step explanation:

Given

Graph h:

[tex](x_1,y_1) = (1,-5)[/tex]

[tex](x_2,y_2) = (9,1)[/tex]

Required

Plot [tex]h^{-1}(x)[/tex]

First, calculate h(x)

Calculate slope (m)

[tex]m = \frac{y_2 - y_1}{x_2 - x_1}[/tex]

[tex]m = \frac{1--5}{9-1}[/tex]

[tex]m = \frac{6}{8}[/tex]

[tex]m = \frac{3}{4}[/tex]

The equation is:

[tex]y = m(x - x_1) + y_1[/tex]

So, we have:

[tex]y = \frac{3}{4}(x - 1) -5[/tex]

[tex]y = \frac{3}{4}x - \frac{3}{4} -5[/tex]

[tex]y = \frac{3}{4}x + \frac{-3 - 20}{4}[/tex]

[tex]y = \frac{3}{4}x - \frac{23}{4}[/tex]

Next, calculate [tex]h^{-1}(x)[/tex]

Swap y and x

[tex]x = \frac{3}{4}y - \frac{23}{4}[/tex]

Solve for y

[tex]\frac{3}{4}y = x + \frac{23}{4}[/tex]

Multiply through by 4

[tex]3y = 4x + 23[/tex]

Divide through by 3

[tex]y = \frac{4}{3}x + \frac{23}{3}[/tex]

Replace y with [tex]h^{-1}(x)[/tex]

[tex]h^{-1}(x) = \frac{4}{3}x + \frac{23}{3}[/tex]

See attachment for graph

El ingreso que perciben en un barrio de la Ciudad de Cuenca durante un día de trabajo los niños y adultos de una familia es: $3 los niños menores de 15 años y $5 los adultos. Si al día reúnen un total de $80 y asisten a trabajar 20 personas, ¿Cuántos de ellos serán niños y cuántos serán adultos? R= 10 niños y 10 adultos

Answers

Answer:

im sorry i took long im not the best at math but since 10 equals r im not sure what you need to know but i think its 3+5r=80 i hope this help

Step-by-step explanation:


Find the value of the trigonometric ratio. Simplify the ratio if possible.
(Hint: You may need to use some pythagorean theorem first!)

Answers

Step-by-step explanation:

Given that,

BC = 8

Ac = 15

We can find AB using the pythagoas theorem.

[tex]AB=\sqrt{AC^2+BC^2}\\\\=\sqrt{15^2+8^2}\\\\AB=17[/tex]

We know that,

[tex]\cos\theta=\dfrac{B}{H}[/tex], B is base and H is Hypotenuse

[tex]\cosB=\dfrac{8}{17}[/tex]

Hence, this is the required solution.

f(2) 42 g(x) = 2x +3 Find 4) (. Include any restrictions on the domain. O A. 2.2---3 417 2 > 0 B. 47 2 2 r--3, (1) (a) = (4) (a) x A 을 c. (5) (r) = 1,2*0 OD (1) (a= + 2​

Answers

Answer:

Option D

Step-by-step explanation:

Given f(x) = [tex]\sqrt[3]{4x}[/tex]

g(x) = 2x + 3

Since, [tex](\frac{f}{g})(x)=\frac{f(x)}{g(x)}[/tex]

                     [tex]=\frac{\sqrt[3]{4x}}{2x+3}[/tex]

This function is defined for the denominator is not equal to zero.

(2x + 3) ≠ 0

x ≠ [tex]-\frac{3}{2}[/tex]

Therefore, Option D will be the correct option.

which measure of central tendency best describes the situation, the ages of 7 people who auditioned for a role in a play: 17,20,22,12,15,12,21

Answers

Answer:

Both the mean and median and they both gives a measure of 17

Step-by-step explanation:

Given the data about the ages of 7 people :

17,20,22,12,15,12,21

Ordered data: 12, 12, 15, 17, 20, 21, 22

To find the median :

Median = 1/2 * (n+1)th term

n = number of observations = 7

Median = 1/2(7+1)th term

Median = 1/2 *8 = 4th term = 17

Mean = Σx / n = (sum of ages) / number of observations

Mean = 119 / 7 = 17

:

Given that ZABC = ZDBE, which statement must be
true?
ZABC - ZABD
ZABD Z CBE
Z CBD , ZDBE
Z CBD - ZABC

Answers

Answer:

B abd = cbe

Step-by-step explanation:

ABC = DBE

so

ABC + CBD = DBE + CBD

The statement which is true is ZABC - ZABD, the correct option is A.

What is a solution to a system of equations? (SOLUTION GRAPHICALLY)

For a solution to be solution to a system, it must satisfy all the equations of that system, and as all points satisfying an equation are in their graphs, so solution to a system is the intersection of all its equation at single point(as we need common point, which is going to be intersection of course)(this can be one or many, or sometimes none)

We are given that;

Angle ABC = Angle DBE

Now,

A flowchart proof is a formal proof that is set up with boxes that flow from one to the next with arrows1. The statements, which are true facts that we know, are placed in the boxes, with the reason we know them on a line underneath2. We can prove theorems are true using flowchart proofs.

To justify each step in the flowchart proof, we have to provide a reason for each statement based on definitions, postulates, properties or previously proven theorems. Here is how we can fill in the blanks:

A: Given B: Definition of angle bisector C: Definition of congruent angles

Therefore, graphically the answer will be ZABC - ZABD.

Learn more about finding the solution graphically here:

https://brainly.com/question/26254258

#SPJ7

What is the probability that out of 250 babies born, 110 or fewer will be boys?
Aaaume that boys and girls are equally probable, and round your answer to
the nearest tenth of a percent,
A. 3.3%
B. 75.7%
C. 28.5%
O D. 97.5%

Answers

Answer:

A. 3.3%

Step-by-step explanation:

We use the normal approximation to the binomial to solve this question.

Binomial probability distribution

Probability of exactly x sucesses on n repeated trials, with p probability.

Can be approximated to a normal distribution, using the expected value and the standard deviation.

The expected value of the binomial distribution is:

[tex]E(X) = np[/tex]

The standard deviation of the binomial distribution is:

[tex]\sqrt{V(X)} = \sqrt{np(1-p)}[/tex]

Normal probability distribution

Problems of normally distributed distributions can be solved using the z-score formula.

In a set with mean [tex]\mu[/tex] and standard deviation [tex]\sigma[/tex], the zscore of a measure X is given by:

[tex]Z = \frac{X - \mu}{\sigma}[/tex]

The Z-score measures how many standard deviations the measure is from the mean. After finding the Z-score, we look at the z-score table and find the p-value associated with this z-score. This p-value is the probability that the value of the measure is smaller than X, that is, the percentile of X. Subtracting 1 by the pvalue, we get the probability that the value of the measure is greater than X.

When we are approximating a binomial distribution to a normal one, we have that [tex]\mu = E(X)[/tex], [tex]\sigma = \sqrt{V(X)}[/tex].

250 babies, boys and girls equally as likely:

This means that [tex]n = 250, p = 0.5[/tex].

Mean and standard deviation:

[tex]\mu = E(X) = np = 250*0.5 = 125[/tex]

[tex]\sigma = \sqrt{V(X)} = \sqrt{np(1-p)} = \sqrt{250*0.5*0.5}[/tex]

Probability that out of 250 babies born, 110 or fewer will be boys?

Using continuity correction, this is P(X < 110 + 0.5) = P(X < 110.5), which is the p-value of Z when X = 110.5. So

[tex]Z = \frac{X - \mu}{\sigma}[/tex]

[tex]Z = \frac{110.5 - 125}{\sqrt{250*0.5*0.5}}[/tex]

[tex]Z = -1.83[/tex]

[tex]Z = -1.83[/tex] has a p-value of 0.033

0.033*100% = 3.3%, so option A.

Two numbers differ by 18. If 2/7 of the larger is 6 more than 1/4 of the smaller, find the two numbers

Answers

Answer:

Step-by-step explanation:

1) x-y=18

2) 2x/7-y/4=6

from 1) x=18+y      substituting in 2)

2(18+y)/7-y/4=6

36/7+y(2/7-1/4)=6

y=(6-36/7)/(2/7-1/4)

y=(6/7)/(1/28)

y=24

x=18+y=18+24

x=42

Question 2 of 25
What do both of these functions have in common?
f(x) = |x-71-2
g(x) = 1.5 In (x - 2) +7
A. They have the same vertical shift
B. They have the same end behavior
C. They have the same vertical stretch
O D. They have the same horizontal translation

Answers

Answer:they have the same vertical stretch

Step-by-step explanation:

explain why triangles in the figure are similar. then find the missing length x

Answers

Answer:

∨∨∨∨see below∨∨∨∨∨∨

Step-by-step explanation:   6  26  18  13

The two outside angles are congruent.  The two inside angles are supplemental thus they are equal.  The last two angles the high one and the lower one must sum to 180° in their respective triangles so they are equal since their similar angles are equal.

find x

4 is to x as 5 is to 7.5

   4/x = 5/7.5      solve for x

   4 × 7.5 / 5  = x

    30 / 5  =  x

       6 = x

The height of a triangle is 7 feet greater than the base. the area of the triangle is 247 ft.² find the length of the base and the height of the triangle

Answers

Answer:

Base = 19

Height = 26

Step-by-step explanation:

Given :

Height, h = base, b + 7 feets

The area of triangle, A = 247 ft²

The area of a triangle is given by :

A = 1/2 * base * height

247 = 1/2 * b * (b + 7)

247 = 0.5 * b * (b + 7)

247 = 0.5b² + 3.5b

0.5b² + 3.5b - 247 = 0

Solving the quadratic equation ; using the quadratic equation solver, the roots are :

b = -26 or b = 19

Length of base can't be negative :

Hence,

Base, b = 19

Height, h = base + 7 = 19 + 7 = 26

Select the correct answer.
AB and BC form a right angle at point B. If A = (-3,-1) and B = (2, 4), what is the equation of BĆ?
O A x + 3y = 16
OB. 2x+y= 12
OC.-7x - 5y=-48
OD. 7x - 5y = 48

Answers

9514 1404 393

Answer:

  x + y = 6   (or choice C, see comment)

Step-by-step explanation:

The equation of the line through point B and perpendicular to AB can be written as ...

  (Bx -Ax)(x -Bx) +(By -Ay)(y -By) = 0 . . . . where A = (Ax, Ay) and B = (Bx, By)

  (2 -(-3))(x -2) +(4 -(-1))(y -4) = 0

  5x +5y -30 = 0

  x + y = 6 . . . . . . reduce to standard form

__

Additional comment

None of the offered choices describes a line through point B. (See second attachment). We note that several of the lines go through the point (4, 4). The segment joining points (-3, -1) and (4, 4) will be perpendicular to the line -7x -5y = -48, answer choice C.

Please talk to your teacher about this question.


PLEASE HELP EASY MATH

Tomas needs lumber to make corner braces for a piece of furniture he is building. He cuts 4 feet from a board that is 8 feet long. With the wood he has left, Tomas makes 18 corner braces of equal size.
How many inches of wood does each brace use? - in​

Answers

18/4, the answer is 4.5, if i understand the problem

Answer:  each brace is 2 & 2/3 inches long

=====================================================

Explanation:

"he cuts 4 feet from a board that's 8 feet long" means that he has 8-4 = 4 feet to work with. This converts to 4*12 = 48 inches.

Next, we cut that into 18 equal pieces

48/18 = 8/3 = 2 & 2/3

This is approximately 2.667 inches

A child's wading pool is 4 feet wide, 6 feet long, and 1 foot deep. how many gallons of water will fill the pool hold?

Answers

Answer:

180 gallons of water

Step-by-step explanation:

4×6×1=24

24×7.5=180

Hope this helps!

Teresa received a $90 gift card for a coffee store. she used it in buying some coffee that cost her $8.21 per pound. After buying the coffee, she had $65.37 left on her card. how many pounds of coffee did she buy?​

Answers

Answer:

3 Pounds of Coffee

Step-by-step explanation:

To find out how much she spent on the pounds of coffee, you would have to takeaway $65.37 away from the total of $90 which will give you $24.63, which is how much she has spent on the coffee. To find out how many pounds of coffee she bought, you would have to divide $24.63 by $8.21 giving you 3, which is how many pounds of coffee Teresa bought.

Much appreciated if this is marked as brainliest :)

Solve the following equation algebraically show the steps:


n/3 -5 = 5

Answers

[tex]\longrightarrow{\green{ \: n = 30 }}[/tex] 

[tex]\large\mathfrak{{\pmb{\underline{\red{Step-by-step\:explanation}}{\red{:}}}}}[/tex]

[tex] \frac{n}{3} - 5 = 5[/tex]

➼ [tex] \: \frac{n}{3} = 5 + 5[/tex]

➼ [tex] \: \frac{n}{3} = 10[/tex]

➼ [tex] \: n = 10 \times 3[/tex]

➼ [tex] \: n = 30[/tex]

Therefore, the value of n is 30.

[tex]\large\mathfrak{{\pmb{\underline{\blue{To\:verify}}{\blue{:}}}}}[/tex]

[tex] \frac{30}{3} - 5 = 5[/tex]

✒ [tex] \: 10 - 5 = 5[/tex]

✒ [tex] \: 5 = 5[/tex]

✒ [tex] \: L.H.S.=R. H. S[/tex]

Hence verified.

[tex]\bold{ \green{ \star{ \orange{Mystique35}}}}⋆[/tex]

Armando works in a research institute and they have collected the following data on a random sample of 500 students,
Gender
GRE score before and after participation in a GRE prep course (GRE is a test you take for entering graduate school)
Armando wants to answer questions the following questions:
1: After taking the GRE preparation course, did women show more improvement on the GRE score or men?
2: After the GRE preparation course, did more women get admitted to the graduate program or men?
3: What statistical method should he use?
a. He should use the one-sample test of the mean to answer question one and the one-sample test of proportion to answer question b.
b. He should use the two-sample test of the mean to answer question one and the two-sample test of proportion to answer question two.
c. He should use the two-sample test of proportion to answer question one and the two-sample test of the mean to answer question b.
d. He should use the paired-sample of the mean to answer question one and two sample test of the mean to answer question two.

Answers

Answer:

b. He should use the two-sample test of the mean to answer question one and the two-sample test of proportion to answer question two.

Step-by-step explanation:

Here we have two independent groups, Men and Women, therefore, to show if it is the Men or Women who performed better, this can be determined based the mean scores of each group. Therefore, use the two sample test of mean which is used to test whether the unknown mean of two independent population are equal or not.

In other to determine those who have the greater number of admissions between the two groups, this is determined by comparing the proportion of each group admitted. Hence, rather Than testing for the mean, we compare the proportion. Hence, we use the two sample test of proportion.

x^(2)+y^(2)+14x+18y+114=0


i will give u brainliest and my eternal love

Answers

Answer:

(x+7)^2+(y+9)^2=16

Step-by-step explanation:

This is the equation written in standard form, I'm not sure if that's what you wanted.

If people send links report them

10)
X + 80
700
B) 8
A) 5
C) -10
D) 7
I

Answers

Answer:

x+80=70°(exterior alternate angles)

x=70-80

x=-10

Other Questions
A certain word game contains a large 20-sided die with letters instead of numbers on the sides. If all of the vowels (a, e, i, o, and u) are on the die, then what is the probability of rolling a vowel? is due today !! need help (show work/explanation) Which country created the first fuel used to propel early rockets?Question 1 options:A) United StatesB) Soviet UnionC) ChinaD) Germany PLEASE look at the picture !! In paragraph 8 of the passage, what does the phrase more than replied suggest about Hollis Meynells character?Question 1.A. She tends to give people things they do not want.B. She is willing to actively pursue her desires.C. she continues to send items other than letters to Lieutenant BlandfordD. She feels more for Lieutenant Blandford than he feels for her.Question 2. Which detail from elsewhere in the story supports the answer in Question 1?A. She sends Lieutenant Blandford a book.B. She admits to Lieutenant Blandford that she is probably unattractive.C. She leaves Lieutenant Blandford in the train station.D. She asks the woman in the brown coat to talk to Lieutenant Blandford. How much interest will be earned on a $1600 investment at 7.72% compoundedquarterly for 9 years? Charles Darwin, Charles Lyell and Alfred Russell Wallace all contributed to the theory ofevolution. Which statement best explains how support for this theory was obtained from thescientific community?Group of answer choicesEach scientists dissected different species from different parts of the world, then compared and shared their findings with each other.Many scientific observations, related to biogeography and comparative anatomy, were made, documented and shared.Together the scientists analyzed and compared cave drawings of various species of prehistoric animals from different parts of the world.Each scientist worked in isolation to experiment on fossils and prove their beliefs regarding evolution. Yes Which equation would have the steepest graph? Explain how you know? DNA uses1. ribose and uracil2. deoxyribose and thymine3. deoxyribose and adenine while RNA uses 1. deoxyribose and thymine 2. ribose and thymine 3. ribose and uracil a 60 kg runner accelerates during a race at 3m/s2. what force is exerted on the earth with each step Explain why the set of polynomials is closed under subtraction in a cave, a stalactite gets 4 millimeters longer each year. This year it is 72 centimeters long. How many years until it is 1 meter long? professional email explanation of why the assignments was late because I'm moving at the moment hi please answer me asap I will mark brainlist F is proportional to a If F = 36 when a = 9 find, a when F = 32 Wilt Chamberlain scored 100 points in a single basketball game on March 2, 1962. In that game, he made 36 field goals (baskets) in 63 attempts. Find the ratio of field goals to attempts Something that can be broken down by living organisms isa.biodegradable.c.ecological.b.economic.d.renewable.Please select the best answer from the choices providedABCD Which statement best explains how images help readers? (1) its no wonder students grades drop when they enter highschool. (2) they are suffering various effects of sleep deprivation The perimeter of a rectangle is 80x. The base is 7 times the height. In terms of x, what are the dimensions of the rectangle?a.h = 10x, b = 10xc.h = 3x, b = 5xb.h = 5x, b = 35xd.h = 5x, b = 25x What is the usual CORRECT pairing of DNA? Look closely at the picture to figure it out.A. T-C and G-AB. C-G and A-T